2011 U OF I MOCK PUTNAM CONTEST
Solutions
1. [Variation of A2, Putnam 1987] The sequence of digits
                                                    1 2 3 4 5 6 7 8 9 1 0 1 1 1 2 1 3 ...
   is obtained by writing out the natural numbers in order. Let f (n) denote the position of the first digit of the number
   n in this sequence. Thus, for example, f (1) = 1, f (2) = 2, f (10) = 10 (since the integer n = 10 occupies positions 10
   and 11 in this sequence), f (11) = 12 (since 11 occupies positions 12 and 13), f (12) = 14 (since 12 occupies positions
   14 and 15, and so on.
   Find, with proof, a simple explicit formula for f (10k ), where k is an arbitrary positive integer.
   Solution. The desired formula is f (10k = k10k  (10k  1)/9 + 1 . To prove this, note that 10k is the first integer
   with k + 1 digits. Thus, the position of its first digit is 1 plus the total number of positions occupied by the integers
   with at most k digits.
   Now there are 10i  10i1 integers with exactly i digits (namely, all integers n in the range 10i1  n < 10i ), so the
   total number of positions occupied by integers with i digits is i(10i  10i1 ). Adding these counts for i = 1, 2, . . . , k  1
   gives the number of positions occupied by integers with at most k digits:
                                              k
                                              X                   k
                                                                  X         k1
                                                                            X
                                                (10i  10i1 )i =   10i i      10j (j + 1)
                                              i=1                        i=1            j=0
                                                                                   k1
                                                                                   X
                                                                       = 10k k          10j
                                                                                   j=0
                                                                                      10k  1
                                                                       = 10k k               .
                                                                                      10  1
   Adding 1 to this count gives the above formula for f (10k ).
               
2. Let an = [( 2 + 1)n ], where [x] denotes the greatest integer  x (i.e., the floor function). Prove that an is even if and
   only if n is odd.
                                           
   Solution. Let bn := (1 + 2)n + (1  2)n . Expanding each of the two nth powers by the binomial theorem shows
   that
                                          n  
                                         X     n       k         k        X n
                                    bn =             ( 2) + ( 2) = 2                  2k/2 .
                                               k                                    k
                                         k=0                                k even
                                           n                                             
   Hence bn is an even integer. Now (1    2)   is a number  of absolute
                                                                         value  at most 2  1 < 1/2 and is negative
                                                                                                                       for n
   odd, and positive for n even. Since ( 2 + 1)n = bn  (1  2)n , by the definition of bn , it follows that [( 2 + 1)n ] is
   equal to bn when n is odd, and equal to bn  1 when n is even. Since bn is always even, this proves the claim.
3. [A2, Putnam 2003] Let a1 , . . . , an , b1 , . . . , bn be positive real numbers. Prove that
                                     (a1 . . . an )1/n + (b1 . . . bn )1/n  (a1 + b1 )1/n . . . (an + bn )1/n .
   Solution. Dividing by the right-hand side, the given inequality can be rewritten as
   (1)                                               (x1 . . . xn )1/n + (y1 . . . yn )1/n  1,
   where
                                                      ai                                 bi
                                                        xi =
                                                           ,                   yi =           .
                                                   ai + bi                            ai + bi
   Applying AGM to each term on the left of (1) gives
                                                                          n               n         n
                                                                       1X         1X         1X
                             (x1 . . . xn )1/n + (y1 . . . yn )1/n          xi +       yi =       (xi + yi ) = 1,
                                                                       n i=1      n i=1      n i=1
   since xi + yi = 1 for each i.
4. Let P1 , P2 , P3 , . . . be a sequence of points in 3-dimensional space satisfying (i) |Pi |  1 for all i and (ii) |Pi Pj |  1
   for all i and j with i 6= j. (Here |P Q| denotes the usual (Euclidean) distance between P and Q, and |P | denotes the
   distance between P and the origin.)
    (a) Prove that, if  > 3, then the infinite series                                                                           
                                                                           X       1
                                                                           i=1
                                                                                 |Pi |
        converges.
    (b) Show that there exists a sequence of points Pi satisfying conditions (i) and (ii) above for which the above series
        diverges when  = 3.
   Solution. (a) First note that, without loss of generality, we may assume that the Pi all lie in the first octant. Subdivide
   this octant into boxes of side length 1/2 of the form                                                                                     
                                                    m m+1         n n+1           p p+1
                                    B(m, n, p) =      ,             ,         ,           ,
                                                    2    2         2    2         2    2
                                                                                                                    p
   where m, n, p are nonnegative integers. Since the largest distance between any two points in such a box is 3/4 < 1,
   such a box can contain at most one of the points Pi . Moreover, since |Pi |  1, the box B(0, 0, 0) cannot contain a point
   Pi .
   Now note that, if Pi  B(m, n, p), then
                                                             m n p  1 p
                                                    |Pi |     , ,   =    m2 + n2 + p2 .                                                                     
                                                               2 2 2     2
   Hence                                     
                 X       1            X                      2                              
                 i=1
                       |Pi |       m,n,p=0
                                                 (m2      + n2 + p2 )/2
                                  max(m,n,p)>0                                         
                                         X            1
                              3!2              2 + n2 + p2 )/2
                                                                                 (using symmetry in m, n, p)
                                      0mnp
                                              (m
                                         p6=0                                            
                                           X          1
                              6  2                        (since m2 + n2 + p2  p2 )
                                                     p
                                        0mnp
                                           p6=0                                         
                                        X   (p + 1)2
                              6  2                        (since there are  (p + 1)2 choices of (m, n) with m, n  p)
                                        p=1
                                               p                                        
                                        X (2p)2
                              6  2                     (since p + 1  2p)
                                       p=1
                                               p                                                                                
                                         X       1
                             = 24  2                 ,
                                         p=1
                                               p2
   and since  > 3, the latter series converges. This proves part (a).
   (b) If we let the Pi be the lattice points (m, n, p) with positive integral coordinates, then the conditions |Pi |  1 and
                                                                       1
  |Pi Pj |  1 are satisfied. On the other hand, we have
                            
            X       1        X            1
                         
            i=1
                  |Pi |3   m,n,p=1
                                   (m2 + n2 + p2 )3/2
                                   k+1       k+1        k+1
                              2X1 2 X1 2 X1
                             X                                                 1                         
                             k=1 m=2k
                                                                   (m2   +    n2   + p2 )3/2
                                             n=2k           p=2k
                                   k+1       k+1        k+1
                              2X1 2 X1 2 X1
                             X                                        1
                                                                                   (since (m2 + n2 + p2 )3/2  (3  22k+2 )3/2  100  23k )
                                                                   100  23k
                             k=1 m=2k        n=2k           p=2k                                 
                              1  X                 1
                         =               (2k )3             (since there are (2k )3 choices of (m, n, p))
                             100                  23k
                                   k=1                                    
                            1 X
                         =      1 = ,
                           100
                                   k=1
                  P
  so the series    i=1   1/|Pi |3 diverges.
5. Determine, with proof, whether the series
                                                                                                                                                    X          1
                                                                              n 2+cos(2 ln n)
                                                                          n=1
  converges.
  Solution.      We claim that the series diverges (though only barely so). The proof is somewhat technical, but the
  underlying idea is easy to describe: Namely, focus on integers for which () ln n  k + 1/2, where k is a positive integer.
  In this case, cos(2 ln n)  cos(2(k + 1/2)) = 1, so the exponent of n in the given series will be approximately 1.
  Thus, if one can show that () holds for long enough intervals of ns, then the series behaves like a harmonic series over
  these stretches and therefore diverges.
  To make this precise, define the intervals Ik , for k = 1, 2, 3, . . . , by
                                                       h                      
                                                 Ik = ek+1/2 , ek+1/2+1/(2k) .
  Note that these intervals are disjoint. We will show that the given sum restricted
                                                                             P      to the interval Ik is  c/k, where c
  is a positive constant. Hence the entire series is bounded from below by  k=1 c/k, a divergent series.
  To prove this, note first that
                                                        1                 1    1
                              n  Ik  ek+ 2  n < ek+ 2 + 2k
                                             1               1    1
                                      k +  ln n < k + +
                                           2          2 2k                            
                                                     1                                 1  1
                                     = cos 2 k +           cos(2 ln n) < cos 2 k + +
                                                     2                                 2 2k
                                                                              
                                                                              1
                                      cos()  cos(2 ln n) < cos  1 +
                                                                              k
                                                                   
                                     = 1  cos(2 ln n) < 1 + ,
                                                                   k
  where the latter inequality follows from the bound
                                     cos((1 + x))  cos() + x max | sin((1 + t))|  1 + x.
                                                                                    0tx
  Also, if n  Ik , then
                                                                               1     1
                                                                   ln n < k +    +      k + 1  2k,
                                                                               2 2k
                                                               1           e(ln n)/k   e2
                                                                         =                   .
                                                             n1+/k            n          n
                                                                                   2
  It follows that
                             X            1            X e2
                                                     >          .
                             nIk
                                    n2+cos( ln n)     nIk
                                                             n
                                                            Z                  
                                                        2       1        1
                                                     e             dt  k+1/2 ,
                                                               I t       e
                                                             k                                     
                                                        2       k+1/2+1/k          k+1/2      1
                                                     =e      ln(e           )  ln(e       )  k+1/2 ,
                                                                                              e
                                                                      
                                                              1
                                                      e2      ek .
                                                              k
   Summing the latter expression over all k gives a divergent series. Hence the given series diverges as well.
6. Let Gn denote the geometric mean of the n binomial coefficients n1 , n2 , . . . , nn .
                                                                                                                        Prove that the limit limn n Gn exists, and find its value. (You may only use the definition of binomial coefficients
   and standard results from calculus, but not, for example, asymptotic formulas for binomial coefficients or factorials.)
                                           1/n    
   Solution. We will show that limn Gn = e. Using the definition of the binomial coefficients, we get
                                                   n               n
                                                   Y  n              Y          n!           n!n
                                         Gnn   =                =                      = 2 2
                                                           k                k!(n  k)!  1! 2! . . . n!2
                                                   k=1               k=1
                                                       1n  2n     nn
                                               = 2n 2n2 2n4                     .
                                                1 2     3       . . . (n  1)4 n2
  Note that, for each i  {1, 2, . . . , n}, the factor i occurs exactly n times in the numerator and 2(n  i + 1) times in the
  denominator. Hence,
                                                               n
                                                               X
                                                   ln Gnn =          (ln i)(n  2(n  i + 1))
                                                               i=1
                                                               Xn                         n
                                                                                          X
  (1)                                                      =         (ln i)(2i  n)  2         ln i.
                                                               i=1                        i=1
  Also,
                                       n                                         
                                      X                             n(n + 1)    2
  (2)                                     (ln n)(2i  n) = (ln n) 2           n = n ln n.
                                      i=1
                                                                       2
  Subtracting (2) from (1) gives
                                                    n
                                                    X                                   n
                                                                                        X
                                       ln Gnn =           (ln i  ln n)(2i  n)  2           ln i + n ln n.
                                                    i=1                                 i=1
  Dividing by n2 we get
                                                                   n              
                                                    1           1X        i     i
  (3)                                ln G1/n
                                         n   =         ln G n
                                                            n =        ln     2    1  + R(n),
                                                    n2          n i=1     n     n
  where
                                                                         n
                                                                ln n   2 X         ln n
                                                     R(n) =           2    ln i +      .
                                                                 n    n i=1         n
  Since
                                                                        n
                                                               ln n   2 X         3 ln n
                                                   |R(n)|          + 2    ln n         ,
                                                                n    n i=1           n
                                                                        3
                                                                                1/n
we have limn R(n) = 0, so the term R(n) has no effect on the limit of ln Gn , and we can therefore ignore it.
                                                                                           R1
The remaining term on the right of (3) can be interpreted as a Riemann sum for the integral 0 (ln x)(2x  1)dx, and
evaluating this integral gives the desired limit:
                                                             n           
                                                          1X        i    i
                                   lim ln G1/n
                                           n   = lim             ln     2 1
                                  n              n n            n    n
                                                            i=1
                                                   Z 1
                                                 =     (ln x)(2x  1)dx
                                                     0
                                                                1 Z 1         1
                                                 = (x2  x) ln x     (x2  x) dx
                                                                
                                                                  0 0          x
                                                     Z 1
                                                                    1
                                                 =      (x  1)dx = .
                                                       0            2
                                 1/n
Exponentiating, we get limn Gn       = e1/2 , as claimed.